2011 AMC 12A Problems/Problem 23
Contents
Problem
Let and
, where
and
are complex numbers. Suppose that
and
for all
for which
is defined. What is the difference between the largest and smallest possible values of
?
Solution 1 (plug and chug)
By algebraic manipulations, we obtain
where
In order for
, we must have
,
, and
.
implies
or
.
implies
,
, or
.
implies
or
.
Since , in order to satisfy all 3 conditions we must have either
or
. In the first case
.
For the latter case note that
and hence,
.
On the other hand,
so,
. Thus
. Hence the maximum value for
is
while the minimum is
(which can be achieved for instance when
or
respectively). Therefore the answer is
.
Shortcut
We only need in
.
Set :
. Since
, either
or
.
so
.
. This is a circle in the complex plane centered at
with radius
since
. The maximum distance from the origin is
at
and similarly the minimum distance is
at
. So
.
Both solutions give the same lower bound, . So the range is
.
Solution 2
note: the "begin{align*}" environment is weird on aops, so if the two things that use it below are duplicated in front of each other, just reload the page or something.
Let
and
for all such
where
is defined. Then either
or
.
This lemma is one that is typically known by people who could make it this far into this AMC 12 anyway (USAMO qualifiers likely).
We plug in
, and equating it to
, we see
\begin{align*}
h\left (h(x)\right ) &=h\left (\frac{ax+b}{cx+d}\right ) \\ &= \frac{a\cdot \frac{ax+b}{cx+d}+b}{c\cdot \frac{ax+b}{cx+d}+d} \\ &= \frac{a(ax+b)+b(cx+d)}{c(ax+b)+d(cx+d)} \\ &= \frac{(a^2+bc)x+(ab+bd)}{(ac+cd)x+(bc+d^2)}=x.
\end{align*}
Then if we clear the denominator, it rearranges into
Since this equation is true for all
(since now the fractions are gone), this implies that the coefficients of this quadratic are all equal to
. Focus on the condition
.
Then all the other coefficients automatically become zero, so is a sufficient condition.
(and
)
Then in , we divide both sides by
to see that
. Similarly, in
, we divide to also see that
. Then since
,
Therefore, in this case, it is required that
, which now works. This proves our desired claim.
Note that we are asking when . So either
, or in the expansion of
, the two corresponding coefficients are negations of each other.
Then , so either
or the two coefficients are negative of each other. Clearly
, since the numerator and denominator are both degree
polynomials. So we would require the coefficients, namely
and
, to be negatives of each other. Then
is a possibility here, giving
.
The two coefficients are negative of each other.
It can be found that \begin{align*}
f(f(x))&=f\left (\frac{z+a}{z+b}\right ) \\ &=\frac{\frac{z+a}{z+b}+a}{\frac{z+a}{z+b}+b} \\ &=\frac{z+a+a(z+b)}{z+a+b(z+b)} \\ &= \frac{(a+1)z+(ab+a)}{(b+1)z+(b^2+a)}.\end{align*} By our lemma, we would require , which rearranges into
. But we are given that
. The triangle inequality for complex numbers states that
, where equality happens when the arguments of
and
are equal. Here, we can see that
but we can also find that
Therefore, because
, then
, giving us
.
When we combine the two cases, we find that the possible range of is
, so the answer is
.
~ethanzhang1001
Video Solution
~MathProblemSolvingSkills.com
Note
This problem is kinda similar to 2002 AIME I Problems/Problem 12
See also
2011 AMC 12A (Problems • Answer Key • Resources) | |
Preceded by Problem 22 |
Followed by Problem 24 |
1 • 2 • 3 • 4 • 5 • 6 • 7 • 8 • 9 • 10 • 11 • 12 • 13 • 14 • 15 • 16 • 17 • 18 • 19 • 20 • 21 • 22 • 23 • 24 • 25 | |
All AMC 12 Problems and Solutions |
These problems are copyrighted © by the Mathematical Association of America, as part of the American Mathematics Competitions.